Hi! Could someone please explain why C is corre...
Thank!
DylanMorris on October 13 at 09:36PM
  • September 1998 LSAT
  • SEC3
  • Q21
1
Reply
Debating A and E
Hi, I debated A and E because I could make an argument that both of them were right. In situat...
Malik on September 17 at 02:44PM
  • September 1998 LSAT
  • SEC3
  • Q24
2
Replies
Wouldn't (E) actually strengthen?
The argument states that individuals who buy cars are spending more money in relation to income. ...
ns120496 on January 23, 2022
  • September 1998 LSAT
  • SEC3
  • Q24
1
Reply
Why B?
Doesn't the part of the stimulus that talks about poets preserving language lend support for B? I...
Alec on August 12, 2021
  • September 1998 LSAT
  • SEC3
  • Q20
2
Replies
Lack of Exercise
Please can you explain how to get to the correct answer here - I picked C! Many thanks, Anna
Anna20 on February 19, 2021
  • September 1998 LSAT
  • SEC3
  • Q7
3
Replies
Please Explain.
Not making the connection with how (E) weakens the argument. Could you please explain? Than...
smilde11 on August 7, 2020
  • September 1998 LSAT
  • SEC3
  • Q24
4
Replies
Q: 20
What type of problem is this?
JoeJune85 on July 22, 2020
  • September 1998 LSAT
  • SEC3
  • Q16
2
Replies
Check on interpretation
Just wanted to get a quick check to see if I'm interpreting the wording in answer choice (C) corr...
Kyle on May 9, 2016
  • September 1998 LSAT
  • SEC3
  • Q10
1
Reply
Request your explanation
Although I got the right answer for this question,I didn't get a pretty much certain logical pict...
Batman on December 3, 2014
  • September 1998 LSAT
  • SEC3
  • Q22
1
Reply